Find the volume V of the solid obtained by rotating the region bounded by the given curves about the specified line.y = 6/7 x2, y = 13/7 − x2; about the x-axis

Answers

Answer 1
The Volume (V) of a solid, obtained by rotating the region bounded by the curve about a specified line; the x-axis

The volume (V) of the solid obtained by rotating the region bounded by the curve y = (6/7)x^2 about the x-axis is V = π[((9/245)b^5) - ((9/245)a^5)].

the volume (V) of the solid obtained by rotating the region bounded by the curve y = (6/7)x^2 about the x-axis is (2/3)π[(13/7) * sqrt(13/7)] cubic units.

To find the volume (V) of the solid obtained by rotating the region bounded by the curve y = (6/7)x^2 about the x-axis, we can use the disk method. Here's a step-by-step explanation:

1. Identify the function: y = (6/7)x^2

2. Determine the limits of integration: Since we are not given specific bounds, let's assume the region is defined between x=a and x=b.

3. Set up the integral for the disk method: V = π∫[f(x)]^2 dx, where f(x) = (6/7)x^2.

4. Substitute the function and limits into the integral: V = π∫[a to b] [(6/7)x^2]^2 dx

5. Simplify the integral: V = π∫[a to b] (36/49)x^4 dx

6. Integrate to x: V = π[(9/245)x^5]∣[a to b]

7. Evaluate the definite integral: V = π[((9/245)b^5) - ((9/245)a^5)] So, the volume (V) of the solid obtained by rotating the region bounded by the curve y = (6/7)x^2 about the x-axis is V = π[((9/245)b^5) - ((9/245)a^5)].

To find the volume V of the solid obtained by rotating the region bounded by the curve y = 13/7 − x2  about the x-axis. Let us consider a region bounded by the curve y = 13/7 − x2 and the x-axis.Let us rotate this region about the x-axis to obtain a solid with a volume.The region bounded by the curve y = 13/7 − x2 and the x-axis is shown below:We need to integrate to find the volume V of the solid obtained by rotating the region bounded by the curve y = 13/7 − x2  about the x-axis. The volume of the solid is given by: V = ∫(π.y2)dx, where y = 13/7 − x2Integrating V, we get: V = π ∫ (13/7 − x2)2 dx Let us evaluate the integralπ ∫ (13/7 − x2)2 dx= π [(13/7) * x - (1/3) * x3] limits from -sqrt(13/7) to +sqrt(13/7))= π [(13/7) * sqrt(13/7) - (1/3) * (13/7)3/2] - π [-(13/7) * sqrt(13/7) - (1/3) * (13/7)3/2]= (2/3)π[(13/7) * sqrt(13/7)] cubic unitsTherefore, the volume of the solid is (2/3)π[(13/7) * sqrt(13/7)] cubic units.

#SPJ11


Related Questions

Please help giving 10 points!
Which statements about the graphs of the equations y = 2x + 4 and y = −x + 4 are true? Select all that apply.

A. The graphs intersect the x-axis at the same point.

B. The graphs intersect the y-axis at the same point.

C. The graphs have different slopes.

D. The graphs are parallel lines.

Answers

Answer:

The answer is B and c.

Step-by-step explanation:

first thing you need to do is  look at slopes

the slopes are different then look at  the y intersects there the same

and finally they both intersect at the same point.

the correct answer is B and C.

A car can travel 33 mi for each gallon of gasoline. The function ​d(x)=33x represents the distance​ d(x), in​ miles, that the car can travel with x gallons of gasoline. The​ car's fuel tank holds 17 gal

Answers

D=1.9411 I believe that’s the answer if I’m wrong pls correct me

find the area of the shaded region!
please solve this with solutions !ASAP​

find the area of the shaded region!please solve this with solutions !ASAP

Answers

the area of the shaded part is 30. 89 cm²

How to determine the area

We have the shape to be a rectangle

The area of the shaded part should be;

Area of rectangle - 2 ( area of a semi circle)

The formula for area of a rectangle

Area = length × width

Area = 12 × 12

Area = 144 cm²

Area of a semicircle = 1/2 πr²

Area = 1/ 2 × 3. 142 × 6²

Area = 56. 56 cm²

Area of shaded part = area of rectangle - 2( area of semicircle)

Area of shaded part = 144 - 2(56. 56)

Area of shaded part = 144 - 113. 11

Area of shaded part = 30. 89 cm²

Thus, the area of the shaded part is 30. 89 cm²

Learn more about area here:

https://brainly.com/question/14137384

#SPJ1

write an equation
(X-P) (x-q)=0

Answers

Answer:

\(x^{2} -pqx+pq\)

Step-by-step explanation:

Medical researchers have determined that for exercise to be beneficial, a person’s desirable heart rate, r, in beats per minute, can be approximated by the formulas r = 143 minus 0.65 a for women r = 165 minus 0.75 a for men, where a represents the person’s age. if the desirable heart rate for a man is 135 beats per minute, how old is he? a. 22.5 years old b. 40 years old c. 45 years old d. 42.5 years old

Answers

Age will be 40 years old.

R, in beats per minute, can be approximated by the formulas, where a represents the person's age.

where R= 143 - 0.65a for women.... (1)

and R= 165 - 0.75a for men,..... (2)

So by implementing these two equation we get a = 40

If the sum of ages is X and Y, and the ratio of their ages is p:q, then the age of Y can be calculated using the formula shown below: Y's age = Y's ratio/Sum of ratios x sum of ages The age dependency ratio is the ratio of dependents (people under the age of 15 or over the age of 64) to the working-age population (people between the ages of 15 and 64). The proportion of dependents per 100 working-age population is shown in the data.

Learn more about Age here

https://brainly.com/question/26423521

#SPJ4

If f(1) = 12, f ' is continuous, and 7 f '(x) dx 1 = 20, what is the value of f(7)? f(7) =

Answers

The value of function f(7) is approximately 14.857.

To find the value of f(7), we can use the information given about f(1), the continuity of f', and the definite integral involving f'.

Let's go step by step:

1. We are given that f(1) = 12. This means that the value of the function f(x) at x = 1 is 12.

2. We are also given that f' is continuous. This implies that f'(x) is continuous for all x in the domain of f'.

3. The definite integral 7 ∫ f'(x) dx from 1 to 7 is equal to 20. This means that the integral of f'(x) over the interval from x = 1 to x = 7 is equal to 20.

Using the Fundamental Theorem of Calculus, we can relate the definite integral to the original function f(x):

∫ f'(x) dx = f(x) + C,

where C is the constant of integration.

Substituting the given information into the equation, we have:

7 ∫ f'(x) dx = 20,

which can be rewritten as:

7 [f(x)] from 1 to 7 = 20.

Now, let's evaluate the definite integral:

7 [f(7) - f(1)] = 20.

Since we know f(1) = 12, we can substitute this value into the equation:

7 [f(7) - 12] = 20.

Expanding the equation:

7f(7) - 84 = 20.

Moving the constant term to the other side:

7f(7) = 20 + 84 = 104.

Finally, divide both sides of the equation by 7:

f(7) = 104/7 = 14.857 (approximately).

Therefore, f(7) has a value of around 14.857.

Learn more about function on:

https://brainly.com/question/11624077

#SPJ11

Mr. beasley 's science class wants to decorate one wall in the classroom like an underwater scene. They use sheets of blue poster board that are 2 feet long and 2 feet wide. How many sheets of blue poster board are used to cover the entire area of the wall? use math words and symbols to explain how you solve

Answers

Answer:

Area of the sheet = 2×2 = 4 feet2

Step-by-step explanation:

54. A bag contains 80 colored tokens. Of all
the tokens in the bag, 25 are black and
5
16 are red.
a. Find, in percent form, the
probability of choosing a black
token.
b. Find, in percent form, the
probability of choosing a red
token.

Answers

The probability of choosing a black token from the bag is 31.25% and the probability of choosing a red token from the bag is also 31.25%.

What is probability?

Probability of an event is the ratio of number of favorable outcome to the total number of outcome of that event.

A bag contains 80 colored tokens. Of all the tokens in the bag, 25 are black and 5/16 are red.

a. In percent form, the probability of choosing a black token.

There are total tokens are 80 in which black token are 25 black tokens. Thus, the probability of choosing black token is,

\(P=\dfrac{25}{80}\\P=0.3125\\P=31.25\%\)

b.In percent form, the probability of choosing a red token.

There are total 5/16 tokens in bag. Thus, the number of red tokens in bag is,

\(n=\dfrac{5}{16}\times80\\n=25\)

Thus, the probability of choosing red token is,

\(P=\dfrac{25}{80}\\P=0.3125\\P=31.25\%\)

Thus, the probability of choosing a black token from the bag is 31.25% and the probability of choosing a red token from the bag is also 31.25%.

Learn more about the probability here;

https://brainly.com/question/24756209

#SPJ2

An author published a book which was being sold online. The first month the author sold 19000 books, but the sales were declining steadily at 7% each month. If this trend continues, how many total books would the author have sold over the first 12 months, to the nearest whole number?

Answers

If this trend continues, the total books the author would have sold over the first 12 months is 157,810 books.

How to calculate the total books sold over the first 12 months?

In this scenario, we would calculate the total books sold by this author over the first 12 months as follows;

First month = 19,000 books.

Second month; 19,000 × (1 - 7)% = 19,000 × 93/100 = 17,670 books.

Third month; 17,670 × (1 - 7)% = 17,670 × 93/100 = 16,433 books.

Fourth month; 16,433 × (1 - 7)% = 16,433 × 93/100 = 15,283 books.

Fifth month; 15,283 × (1 - 7)% = 15,283 × 93/100 = 14,213 books.

Sixth month; 14,213 × (1 - 7)% = 14,213 × 93/100 = 13,218 books.

Seventh month; 13,218 × (1 - 7)% = 13,218 × 93/100 = 12,293 books.

Eigth month; 12,293 × (1 - 7)% = 12,293 × 93/100 = 11,432 books.

Ninth month; 11,432 × (1 - 7)% = 11,432 × 93/100 = 10,632 books.

Tenth month; 10,632 × (1 - 7)% = 13,218 × 93/100 = 9,888 books.

Eleventh month; 11,432 × (1 - 7)% = 11,432 × 93/100 = 9,196 books.

Twelveth month; 9,196 × (1 - 7)% = 9,196 × 93/100 = 8,552 books.

Next, we would add all of the books sold in each month together;

Total books sold = 19,000 + 17,670 + 16,433 + 15,283 + 14,213 + 13,218 + 12,293 + 11,432 + 10,632 + 9,888 + 9,196 + 8,552

Total books sold = 157,810 books.

Read more on total books here: https://brainly.com/question/18306224

#SPJ1

–81, 108, –144, 192,. Which formula can be used to describe the sequence?.

Answers

The formula that can be used to describe the sequence is:\(a(n) = (-1)^(n+1) * 3^(n) * 4.\)

The given sequence is -81, 108, -144, 192.

The formula that can be used to describe the sequence is:  \(a(n) = (-1)^(n+1) * 3^(n) * 4\), where n is the nth term in the sequence.

This formula is a geometric sequence formula that can be used to describe the given sequence.

The formula represents the nth term of the sequence as a function of the position of the term in the sequence.

Here, n represents the position of the term in the sequence

.For the given sequence, the first term is -81, which corresponds to the first position in the sequence (n = 1).

The second term is 108, which corresponds to the second position in the sequence (n = 2).

The third term is -144, which corresponds to the third position in the sequence (n = 3).

The fourth term is 192, which corresponds to the fourth position in the sequence (n = 4).

Therefore, the formula that can be used to describe the sequence is: \(a(n) = (-1)^(n+1) * 3^(n) * 4.\)

Know more about sequence  here:

https://brainly.com/question/7882626

#SPJ11

\text{The sum of 7 and the cube of a number.}
The sum of 7 and the cube of a number.

Answers

Answer:

Step-by-step explanation:

Lets call the number n.

So the cube of this number is \(n^3\) and the sum of 7 and the cube of this number is \(7+n^3\)

The Zearn Fair charges $1.75 for entrance and $1.05 for each ride. Mr. Sawicki brings $20 with him to the fair. What is the greatest number of rides Mr. Sawicki can go on if he uses $2.50 for food? How much money can Mr. Sawicki spend in total

Answers

Step-by-step explanation:

So

Total money = 20

Entrance fee = 1.75

Food = 2.50

Ride = 1.05

Amount spend = 1.75 + 2.05 = 3.8

Left for rides = 20 - 3.8 = 16.2

So rides = 16.2 ÷ 1.05 = 15

I think atleast 15 rides

What is the quotient of (x3 - 3x2 + 5x – 3) = (x - 1)?
x2 - 2x - 3
x2 + 2x + 7
x2-3x + 8
x2 - 2x + 3

Answers

Note: Consider "÷" sign instead of "=".

Given:

\((x^3-3x^2+5x-3)\div (x-1)\)

To find:

The quotient.

Solution:

We have,

\((x^3-3x^2+5x-3)\div (x-1)\)

It can be written as

\(=\dfrac{x^3-3x^2+5x-3}{x-1}\)

Splitting the middle terms, we get

\(=\dfrac{x^3-x^2-2x^2+2x+3x-3}{x-1}\)

\(=\dfrac{x^2(x-1)-2x(x-1)+3(x-1)}{x-1}\)

Taking out the common factor (x-1), we get

\(=\dfrac{(x-1)(x^2-2x+3)}{x-1}\)

Cancel the common factors.

\(=x^2-2x+3\)

The quotient is \(x^2-2x+3\).

Therefore, the correct option is D.

Answer:

D

Step-by-step explanation:

write the percent and the decimal shown by the model


percent: __ %


decimal: __


please please please help meee :>

write the percent and the decimal shown by the model percent: __ %decimal: __ please please please help

Answers

The percentage represented is 72%, and the decimal is 0.72

How to write the percent and the decimal?

First lets find the decimal, it will be equal to the quotient between the total number of shaded squares and the total number of squares.

There are 200 squares in total, and of these 200, there are 144 shaded ones, then the decimal is:

d = 144/200  = 0.72

To find the percentage, you only need to multiply the decimal by 100%, we will get:

p = 100%*0.72 = 72%

Learn more about percentages by reading:

https://brainly.com/question/843074

#SPJ1

Write each phrase or sentence as an algebraic expression or equation
6 more than y
3 times a number
5 less than the product of 10 and c
14 more than 6 times t
The sum of x and y is 18
A number minus 12 is y

Answers

6 more than y :    

                      y + 6

3 times a number:

x ← a number    

                           3x

5 less than the product of 10 and c :

10c   ← the product of 10 and c

                                         10c - 5

14 more than 6 times t :

6t  ← six times t

                        6t + 14

The sum of x and y is 18:

x+y    ← the sum of x and y

                                  x + y = 18

A number minus 12 is y:

x - a number

                   x - 12 = y              

The graph of a quadratic function with vertex (1,-1) is shown in the figure below. Find the domain and the range. Write your answers as inequalities, using or as appropriate. Or, you may instead click on "Empty set" or "All reals" as the answer.

Answers

The domain of the function is all real numbers and  range is  y ≥ -1.

Since the vertex is at (1,-1), the axis of symmetry is x = 1.

This means that the domain of the function is all real numbers.

To find the range, we need to consider the y-values of the graph. Since the vertex is the lowest point of the graph, the range must be all y-values greater than or equal to -1.

However, since the parabola opens upwards, there is no upper bound on the y-values.

Therefore, the range is given by y ≥ -1.

Hence, the domain of the function is all real numbers and  range is  y ≥ -1.

To learn more on Functions click:

https://brainly.com/question/30721594

#SPJ1

Which graph represents the system of inequalities?

y+2<5x
y+3x>4
y≥2x

Which graph represents the system of inequalities?y+2&lt;5xy+3x&gt;4y2x

Answers

Answer:

2nd graph above the one you chose

Ray purchased a jacket with 60% discount, for $240. how would you calculate price before a sale?

Answers

A (original cost of the jacket) x 60% = 240
240/ 60% = A

Answer:

$400

Step-by-step explanation:

We will make this question into a function by setting 'p' as the initial price before sale and converting the percent to a decimal (60*0.01).

Because the final price was calculated by multiplying the initial price by 60%, we can represent that with the function:

0.6p=240

All we need to do now is isolate p by dividing out 0.6.

p=400

Giving us an answer of $400 (that's an expensive jacket )

O Assignment is
eBookwork
Consider the conjecture, "Any number divisible by 2 is also divisible by 4." Is each of the numbers 12, 19, 22, 28, and 30 a counterexample of the conjecture?
Complete the following table.

O Assignment iseBookworkConsider the conjecture, "Any number divisible by 2 is also divisible by 4."

Answers

Please find attached a diagram with the completed table

In order to determine which number is  a counterexample of the conjecture, we are to divide each number by 2 and 4. A counterexample of this conjecture would only be divisible by either 2 or 4 but not both of them

Dividing the numbers by 2

12 / 2 = 6

19 / 2 = 9.5

22 / 2 = 11

28 / 2 = 14

30 / 2 = 15

Dividing the numbers by 4

12 / 4 = 3

19 / 4 = 4.75

22 / 4 = 5.5

28 / 4 = 7

30 / 4 = 7.5

22 and 30 are counterexamples because they are divisible by 2 but not by 4

A similar question was solved here: https://brainly.com/question/6698659?referrer=searchResults

O Assignment iseBookworkConsider the conjecture, "Any number divisible by 2 is also divisible by 4."

Does anyone know this?

Does anyone know this?

Answers

Answer:

-6

Step-by-step explanation:

Hello!

Since we are finding f(-1) we put -1 in for x

2(-1)^2 + 8(-1)

Follow PEMDAS

2 * 1 + 8(-1)

2 * 1 + -8

2 + -8 = -6

The answer is -6

Hope this helps!

19. Please help!!! Reporting all fake answers!:-(

19. Please help!!! Reporting all fake answers!:-(

Answers

my answer is (a) I think

CAN YOU GUYS PLEASE ANSWER THE QUESTION RIGHT FOR ONCE

CAN YOU GUYS PLEASE ANSWER THE QUESTION RIGHT FOR ONCE

Answers

Answer:

Step-by-step explanation:

1000*PI*4/3

4000/3*PI

1333.3 (3 REPEATING)*PI

4186.6 (6 REPEATING)

BRAINLIEST PLSSSSSSSSSSSSS

Answer:

4186.67

Step-by-step explanation:

volume of sphere = 4πr³/3

4 × 3.14 × 10 × 10 × 10 ÷ 3 = 4186.67cm³

Can someone tell me the answer to this please?????

Can someone tell me the answer to this please?????

Answers

Answer:

A

Step-by-step explanation:

The triangle NLM has corresponding angles with triangle LND

HELP WHOEVER IS GOOD AT ALGEBRA

Create a polynomial function that
- 6th, 7th, 8th, OR 9th-degree polynomial.
- It has at least two x-intercepts with different multiplicities.
- It has imaginary zeros.
- It has a negative y-intercept.

Answers

if i am correct
f(x)= x^3-6x^2-16x
=x(x-8) (x+2)
zeros : 0,8,-2

a store sells 2 pounds of tomatoes for $7. Write an equation that represents the cost, c, for t pounds of tomatoes

Answers

Answer:

t pounds = 3.5 (or just t = 3.5c)

Step-by-step explanation:

2 pounds of tomatos for 7$ means that one pound of tomatos for 3.5$. Therefore, t pounds = 3.5 c

The equation which represents the sales of tomatoes will be ( t = 3.5c).

What is an expression?

Expression in maths is defined as the collection of numbers variables and functions by using signs like addition, subtraction, multiplication, and division.

Numbers (constants), variables, operations, functions, brackets, punctuation, and grouping can all be represented by mathematical symbols, which can also be used to indicate the logical syntax's order of operations and other features.

2 pounds of tomatoes for 7$ means that one pound of tomatoes for 3.5$. Therefore, t pounds = 3.5 c

Let t represent tomatoes and c represents cost then the expression will be written as below:-

t = 3.5c

Therefore, the equation which represents the sales of tomatoes will be ( t = 3.5c).

To know more about an expression follow

brainly.com/question/723406

#SPJ2


Solve each proportion.
(z-1)/3 = 8/(z+1)

Answers

\(\dfrac{z-1}3 = \dfrac8{z+1}\)

Eliminate the denominators.

\(3(z+1)\cdot\dfrac{z-1}3 = 3(z+1)\cdot\dfrac8{z+1}\)

\(z^2 - 1 = 24\)

Solve for \(z\).

\(z^2 = 25\)

\(\boxed{z = \pm5}\)

what is the volume of a triangular pyramid that is 5 feet tall and has a base area of 9 square feet

Answers

The volume of the triangular prism as described is; 15 cubic foot.

What is the volume of the triangular pyramid?

A triangular pyramid simply refers to any geometric solid with a triangular base, and all three lateral faces are also triangles with a common vertex

It follows from the formula for calculating the volume of a triangular prism that;

Volume = (1/3) × base area × height.

Consequently,

volume of the prism is; = (1/3) ×9 × 5

Volume = 15 cubic foot

Therefore, volume of the triangular prism as described is; 15 cubic foot.

Read more on triangular pyramid:

https://brainly.com/question/12807315

#SPJ1

Factorise fully 18g - 15​

Answers

Answer:

3(6g - 5)

Step-by-step explanation:

1. First, find the GCF of 15 and 18.

Factors of 15:

1, 3, 5, 15

Factors of 18:

1, 2, 3, 6, 9, 18

The GCF of 15 and 18 is 3.

2. Factorize

3(6g - 5)

3. Check you answer:

3(6g) + 3(-5)

18g - 15

Hope this helps!

A fruit seller bought a crate of apples for $66.50.After selling each apple for 55 cents,he was able to cover his cost but did not make a profit of more than $20.How many apples could he have sold?

I have to solve this using simultaneous linear inequalities.

Answers

\(\large{\mathcal{ANSWER:}}\)

Let initially he had x No. of apples

ATQ :-

x-40% of x=420

x-2/5x=420

=> 3x=420×5

x= 700

\(\underline{\boxed{\purple{\tt{©⚘MissyRiel}}}}\)

Hope this helps!

The average of six numbers is -3.65.If five numbers are each -3.20,what is the sixth number.

Answers

we know that the average for six numbers can be writen like:

\(A=\frac{x_1+x_2+x_3+x_4+x_5+x_6}{6}\)

So if 5 of them are -3.20 and the average is -3.65 we can replace that values and find the missing one:

\(-3.65=\frac{5(-3.20)+x}{6}\)

and we solve for x so:

\(\begin{gathered} -3.65\cdot6=-16+x \\ -22+16=x \\ -6=x \end{gathered}\)

So the missing value is -6

Other Questions
TRUE/FALSE.the fundamentals in a campaign include things such as the quality of a candidate's background and how he or she does on the campaign trail. Which of the following laboratory findings should the clinician expect in a patient with untreated Graves' disease?a. Elevated TSHb. Elevated T4c. Elevated thyrotropin-releasing hormone (TRH)d. All of the above Pls help me I only have 45 min and am stuck on this question?!At 6 am the temperature was -5F. By noon the temperature had increased 12. What was the temperature at noon? La cancin Guantanamera es sobre un hombre que...? Three-fifths of the employees at Mac's DiscountGrocery are female and one-fourth of theemployees are under the age of 18. If one-thirdof the female employees are under the age of 18,what is the ratio of males over the age of 18 tomales under the age of 18 who work at Mac'sDiscount Grocery?A) 7:4B) 3:1C) 4:1D) 7:1 Classify each polynomial by degree and by number of terms. Simplify first if necessary. (2a - 5) (a - 1) Select the two values of x that are roots of this equation. the phenotype of an organism Environmental forces that shape the nature of an organization's actions include social, economic, technological, competitive, and ______ forces. Figure 4 A show a joining process traditionally applied to metals, in which molten filler metal alloy usually made of tin and lead which is melted using a hot iron can be drawn or distributed by capillary action into the joint. Rajah 4 A menunjukkan proses penyambungan yang digunakan secara tradisional pada logam, di mana cecair logam aloi pengisi kebiasaannya diperbuat daripada timah dan plumbum yang dicairkan menggunakan besi panas boleh diagihkan melalui tindakan kapilari ke dalam sambungan. Figure/Rajah 4A : Joining Process i. Explain two (2) situations this process is preferable to be used as compared with fusion welding. Terangkan dua (2) situasi proses ini lebih baik digunakan berbanding dengan kimpalan lakuran. ii. State temperature use and list one (1) advantage compared with welding process. Nyatakan penggunaan suhu dan senaraikan satu (1) kelebihan berbanding dengan proses kimpalan. (4 marks/markah) NEED HELP ASAP PLEASE !!! Trace amounts of oxygen gas can be "scrubbed" from gases using the following reaction: 4 Cr2+(aq) + O2(g) + 4 H+(aq)-4 Cr3+(aq) + 2 H2O(l) Which of the following statements is true regarding this reaction? A. O2 (g) is reduced B. Cr2+(aq) is the oxidizing agent. C. O2(g) is the reducing agent. D. Electrons are transferred from 02 to Cr2- A mattress store received a shipment of ten mattresses. Before the shipment, it had 782 mattresses in stock. How many mattresses does the store have now? A tick feeding on a human is an example of Two different formulations of an oxygenated motor fuel are being tested to study their road octane numbers. Sample 1 of size nu = 11 from the first formulation, and Sample 2 of size n2 = 18 from the second formulation have been taken. For Sample 1 the sample mean equals X1 and the sample standard deviation is 51. For Sample 2 the sample mean equals xa and the sample standard deviation is 52. It is known that the population variances of the two formulations are equal. , Use your personally assigned sample means and sample standard deviations (X1, X2 , S1, S2) listed next to the last digit of your student ID in the second page of this document to answer the following questions. (a) Which test statistic would you use to perform a statistical test for the difference in mean road octane number? Explain your answer and state any necessary assumptions. [10 points) (b) Test the hypothesis that the formulation 1 produces a higher road octane number than formulation 2, at the significance level a=0.01 using a fixed-level (critical value) test. State clearly all the steps of the hypothesis testing procedure (the "seven-step procedure"). What is the conclusion of the test in the context of the problem? [15 points) (c) Provide bounds for the P-value of the test using the t-distribution table (not software). Explain the meaning of the calculated P-value and how it can be used to do the hypothesis test of part (b). [10 points) (d) Construct the appropriate one-sided confidence bound for the difference in the mean yields from the two processes to conduct the hypothesis test of part (b) at a=0.05. Use the confidence bound to conduct the test and state your conclusion in the context of the problem. [15 points) 2. Derive the cost function curve for a hypothetical physician practice. How does one optimise the cost of capital and labour in the production of health care? Give an illustration. [10 Marks] Find x. (Hint: Draw an auxiliary line.) 2 describe how core counselling skills can be used in a counselling relationship and in other helping activities. one of the leaders of the free speech movement at the university of california at berkeley was richard nixon. his experience as a leader of the movement gave nixon the national exposure he needed to launch his presidential campaign. (True or False) Can someone help me with these question as soon as possible I will give the brainliest